Question

In the Bertrand model with product differentiation, suppose that the two Bertrand firms face the following...

In the Bertrand model with product differentiation, suppose that the two Bertrand firms face the following symmetric demand curves:

q1 = 96 - 2p1+1/2 p2

q2 = 96 - 2p2 + 1/2p1

where q1, q2 ≥ 0 and p1, p2 ≤ 48. MC for both firms is 12.

  1. Is product differentiation more or less significant in this example than in the example given in the text in Equations 10.3A and 10.3B? Why?
  2. Find the Bertrand equilibrium.

Homework Answers

Answer #1

In class, we considered Cournot competition where two firms choose quantities and let the price be fixed by the market. We consider here a different model of competition on prices called Bertrand competition. Consider 2 firms producing identical products (i.e., that are perfect substitutes) with a constant marginal cost c. Each firm i choose the price pi ∈ [0, 1]. We assume that the demand curve is linear so that the total demand is Q(p) = 1 − p where p = min(p1, p2) and is entirely directed to the firm with the smallest price. If both prices are equal, the demand is equally shared between the two firms. We denote by qi the demand to firm i. For instance, the demand to firm 1 is q1(p1, p2) =    1 − p1 if p1 < p2, 0 if p1 > p2, (1 − p1)/2 if p1 = p2.

Know the answer?
Your Answer:

Post as a guest

Your Name:

What's your source?

Earn Coins

Coins can be redeemed for fabulous gifts.

Not the answer you're looking for?
Ask your own homework help question
Similar Questions
Consider a simplified model of horizontal product differentiation. In class we noted that with quadratic “transportation...
Consider a simplified model of horizontal product differentiation. In class we noted that with quadratic “transportation costs” the demand curve facing an individual firm was decreasing in its own price and increasing in the price of its rival. Suppose an industry has two firms with constant and symmetric marginal costs c1 = c2 = 6. Suppose given levels of product differentiation and transportation costs, demand for firm 1 and firm 2 is: q1 = 60 − 2p1 + p2 q2...
Two firms exist in a market. Demand for firm 1’s product is Q1 = 100 –...
Two firms exist in a market. Demand for firm 1’s product is Q1 = 100 – p1 + ½ p2 Demand for firm 2’s product is Q2 = 100 – p2 + ½ p1 What tells an economist that these two products are substitutes for each other? What tells an economist that these two products are not perfect substitutes? What model would you recommend using to solve for p1 and p2?
Question 2 Consider the following Bertrand game involving 2 firms producing differentiated products. Firms have no...
Question 2 Consider the following Bertrand game involving 2 firms producing differentiated products. Firms have no costs of production. Firm 1’s demand is q1 = 1-p1 + bp2, where b > 0. A symmetric equation holds for firm 2’s demand. a. Solve for the NE of the simultaneous price-choice game b. Compute the firms’ outputs and profits. c. Represent the equilibrium on a best-response function diagram. Show how an increase in b would change the equilibrium.
Consider a market with two identical firms. The market demand is P = 26 – 2Q,...
Consider a market with two identical firms. The market demand is P = 26 – 2Q, where Q = q1 + q2. MC1 = MC2 = 2. 1. Solve for output and price with collusion. 2. Solve for the Cournot-Nash equilibrium. 3. Now assume this market has a Stackelberg leader, Firm 1. Solve for the quantity, price, and profit for each firm. 4. Assume there is no product differentiation and the firms follow a Bertrand pricing model. Solve for the...
1) Which of the following is true of the Symmetric Bertrand model of a duopoly? a)...
1) Which of the following is true of the Symmetric Bertrand model of a duopoly? a) Each firm matches the price increases by the other firm. b) The firm that sets the lower price claims the entire market. c) The total output supplied by the firms determines the market price. d) Firms compete on multiple dimensions like quantity, price, and advertising. e) The demand curve facing an individual firm is kinked at the market price. Which of the following is...
Two Cournot firms produce slightly different products. Product prices depend on both firms' outputs and are...
Two Cournot firms produce slightly different products. Product prices depend on both firms' outputs and are determined by the following equations P1 = 70 - 2Q1 - Q2, P2 = 100 - Q1- 2Q2. Both Firm 1 and Firm 2 have constant marginal cost of $10 and zero fixed cost. Firm 1 chooses Q1 and Firm 2 chooses Q2. (3pts) Find Firm 1's best response as a function of Firm 2's output Q2.   (3pts) Find Firm 2's best response as...
Consider the following variant of the Bertrand Model of Duopoly. Suppose there are two firms producing...
Consider the following variant of the Bertrand Model of Duopoly. Suppose there are two firms producing the same good and they simultaneously set prices for their product. If firm i sets a price pi and firm j sets a price pj, the total quantity demanded for firm i’s product is given by: qi= 10–pi+ ½ pj Each firm produces exactly the qi demanded by the market. Both firms have the same marginal cost of production: c=4. For example, if a...
Suppose two identical firms are in Bertrand Competition with the following market demand and marginal costs...
Suppose two identical firms are in Bertrand Competition with the following market demand and marginal costs P = 124 − 6Q MC = 4 1 Assuming both firms collude what would the price, quantities and (one period) profits be? 2 Assume both firms are colluding to raise the equilibrium price. If one firm defected from (i.e. broke) their agreement how much would they earn? (Assume the game was played once.) 3 Now assume the game is infinitely repeated and the...
Suppose two firms are competing in prices (Bertrand) in an industry where demand is P=360-12Q. Assume...
Suppose two firms are competing in prices (Bertrand) in an industry where demand is P=360-12Q. Assume neither firm faces any fixed costs. (a) If both firms have MC=150, what is the equilibrium price? Profits? (b) Suppose Firm 1 has MC1 = 240 and Firm 2 has MC2 = 0. Approximately how much profit does each firm make? (c) Suppose Firm 1 has MC1 = 204 and Firm 2 has MC2 = 96. Approximately how much profit does each firm make?
Two firms compete by choosing price. Their demand functions are Q1 = 20 - P1 +...
Two firms compete by choosing price. Their demand functions are Q1 = 20 - P1 + P2 and Q2 = 20 +P1 -P2 where P1 and P2 are the prices charged by each firm, respectively, and Q1 and Q2 are the resulting demands. Note that the demand for each good depends only on the difference in prices; if the two firms colluded and set the same price, they could make that price as high as they wanted, and earn infinite...
ADVERTISEMENT
Need Online Homework Help?

Get Answers For Free
Most questions answered within 1 hours.

Ask a Question
ADVERTISEMENT